Natalie and Warren started swimming laps in the pool at 5:47. Natalie finished swimming her laps 27 minutes later. Warren finished swimming his laps 8 minutes after Natalie. What time did Warren finish his laps?

Answers

Answer 1

Answer:

6:22

Step-by-step explanation:

original time minutes when they both started - 47

47+ natalie's time

47+27 =74

minus 60 minutes to account for the hour (6 o'clock)

74-60=14

natalie finished at 6:14

warren finished 8 minutes after natalie's time

14+8=22

the hour did not finish so it will remain at 6

which leaves your final answer for the time that warren finished as 6:22


Related Questions

Mark 4/9 and -7/9 on a number line.​

Answers

where’s the mf number line

Answer:

Step-by-step explanation:

4/9 on the number line would be 0.44.

-7/9 on the number line would be -0.77

the sum of ages of two brothers A and B is 35 A is two thirds of B's age find their ages​

Answers

Answer:

let the ages of the brothers be 2x and 3x

2x +3x=35

5x=35

x=35÷5

x=7

now,

A=2x=2×7=14

B=3x=3×7=21

this is the answer

Factorize x2 + 7x + 12 using middle term break method.

Answers

Answer:

(x + 4)(x + 3)

Step-by-step explanation:

x^2 + 7x + 12

In Quadratic Factorization using Splitting of Middle Term which is x term is the sum of two factors and product equal to last term. Factor each pair by finding common factors.

so we need two number that gives 12 when multiplied and 7 when added

4 and 3 are the numbers because 4*3 = 12 and 4+3 = 12

=x^2 + (4 + 3)x + 12

=x^2 + 4x + 3x + 12

=x(x + 4) + 3(x + 4)

take (x + 4) as common

=(x + 4) (x*1 + 3*1)

=(x + 4)(x + 3)

Help pls I'll give 25 points

Answers

Answer:

[tex]\text{C. }60[/tex]

Step-by-step explanation:

Question from image:

"The digits 1, 2, 3, 4, and 5 can be formed to arrange 120 different numbers. How many of these numbers will have the digits 1 and 2 in increasing order? For example, 14352 and 51234 are two such numbers."

Let's start by taking a look with the number 1. There are four possible places 1 could be, because there needs to be space for the 2 after it.

Checkmarks mark where the 1 can be, the x marks where it cannot be.

[tex]\underline{\checkmark}\:\underline{\checkmark}\:\underline{\checkmark}\:\underline{\checkmark}\:\underline{X}[/tex]

Let's start with the first position:

[tex]\underline{\checkmark}\:\underline{\#}\:\underline{\#}\:\underline{\#}\:\underline{\#}[/tex]

There are four places the 2 can be. For each of these four places, we can arrange the remaining 3 digits in [tex]3!=6[/tex] ways. Therefore, there are [tex]4\cdot 6=\boxed{24}[/tex] possible numbers when 1 is the first digit of the number.

Continue this process with the remaining possible positions for 1.

Second position:

[tex]\underline{X}\underline{\checkmark}\:\underline{\#}\:\underline{\#}\:\underline{\#}[/tex]

There are three places the 2 can be, since the 2 must be behind the 1. For each of these three places, the remaining 3 digits can be arranged in [tex]3!=6[/tex] ways. Therefore, there are [tex]3\cdot 6=\boxed{18}[/tex] possible numbers when 1 is the second digit of the number.

The pattern continues. Next there will be 2 places to place the 2. For each of these, there are [tex]3!=6[/tex] ways to rearrange the remaining 3 digits for a total of [tex]2\cdot 6=\boxed{12}[/tex] possible numbers when 1 is the third digit of the number.

Lastly, when 1 is the fourth digit of the number, there is only 1 place the 2 can be. For this one place, there are still [tex]3!=6[/tex] ways to rearrange the remaining three numbers. Therefore, there are [tex]1\cdot 6=\boxed{6}[/tex] possible numbers when 1 is the fourth digit of the number.

Thus, there are [tex]24+18+12+6=\boxed{60}[/tex] numbers that will have the digits 1 and 2 in increasing order, from a set of 120 five-digit numbers created by the digits 1 through 5, where no digit may be repeated.

Help ASAP!!!!!!!!!!!!!!

Answers

the answer to your question is C

mmmmm hey can you explain my math?​

Answers

Answer:On June 17

the explanation is in the picture

There is a bag filled with 5 blue and 6 red marbles.
Marble is taken at random from the bag, the colour is noted and then it is replaced.
Another marble is taken at random.
What is the probability of getting 2 reds?
thanks,

Answers

Answer:

[tex](\frac{6}{11} )*(\frac{6}{11} )[/tex] = 36/121 =.297

Step-by-step explanation:

Answer:

36/121

Step-by-step explanation:

We have 5+6 = 11 marbles

P(red) = number of red/ total = 6/11

We replace the marble so we still have 11 marbles

P(red) = number of red/ total = 6/11

P(red, red) = 6/11*6/11 = 36/121

In 2 Year 6 classes, 2/5 of the children are girls. There are 39 boys. How many children are there in the class?

Answers

Second time I’ve seen this problem

The distance required to stop a car varies directly as the square of its speed.if 250 feet are required to stop a car traveling 60 miles per hour, how many feet are required to stop a car traveling 96miles per hour​

Answers

Answer:

640 feet.

Step-by-step explanation:

Let d represent the distance required to stop and let s represent the speed of the car.

The distance required to stop varies directly as the square of its speed. In other words:

[tex]d=ks^2[/tex]

Where k is the constant of variation.

250 feet are required to stop a car traveling 60 miles per hour. Substitute:

[tex](250)=k(60)^2[/tex]

Simplify and solve for k:

[tex]\displaystyle 3600k=250\Rightarrow k=\frac{250}{3600}=\frac{25}{360}=\frac{5}{72}[/tex]

So, our equation is:

[tex]\displaystyle d=\frac{5}{72}s^2[/tex]

Then the distance required to stop a car traveling 96 miles per hour will be:

[tex]\displaystyle d=\frac{5}{72}(96)^2=\frac{5}{72}(9216)=640\text{ feet}[/tex]

PLEASE ANSWER ASAP
expand and simplify the following:
a) (√6+√3)(√6-√3) b)(√5-√2)(√2+√5)

Answers

Answer:

[tex]{ \bf{a).}} \\ { \tt{( \sqrt{6} + \sqrt{3} )( \sqrt{6} - \sqrt{3}) }} \\ { \tt{ = ( { \sqrt{6} )}^{2} - {( \sqrt{3}) }^{2} }} \\ { \tt{ = 6 - 3}} \\ { \tt{ = 3}} \\ \\ { \bf{b).}} \\ { \tt{( \sqrt{5} - \sqrt{2})( \sqrt{2} + \sqrt{5} )}} \\ { \tt{ = ( { \sqrt{5} }^{2}) - { (\sqrt{2} }^{2}) }} \\ { \tt{ = 5 - 2}} \\ { \tt{ = 3}}[/tex]

Answer:

(6-3)(5-2)=3×3=9

Step-by-step explanation:

[tex]x ^{2} - {y}^{2} = (x - y)(x + y)[/tex]

[tex]( \sqrt{6} - \sqrt{3} )( \sqrt{6} + \sqrt{3} ) = ( { \sqrt{6} }^{2} - { \sqrt{3} }^{2} ) = 6 - 3 = 3[/tex]

Please Helppp!! 50 Points

Compute the sum

Answers

10235

I just typed it in a calculator

the answer is in the above image

Answer:

Step-by-step explanation:

5* (2⁰ + 2¹ + 2 ²+ 2³+ ........+ 2¹⁰)

Sum of geometric series = [tex]a*\frac{r^{n}-1}{r-1}[/tex]

a -> first term = 1

r- ratio = 2

n --> number of terms = 11

5* (2⁰ + 2¹ + 2 ²+ 2³+ ........+ 2¹⁰)=

 [tex]5*[1*\frac{2^{11}-1}{2-1}]\\\\=5 *[1*(2048-1)]\\\\= 5 * 2047\\= 10235[/tex]

Abdi created a factor rainbow for the number 59.

Answers

Answer:

59 is prime ... 1 & 59 are the only factors

Step-by-step explanation:

 

please help it asks: the graph below represents which of the following functions

Answers

Please comment with a picture of the graph!

The weight Wkg of a metal bar varies jointly as it's length L and the square of it's diameter D mm. If W=140 when D =4 and L=54, find D interm of W and L

Answers

Answer:

D = [tex]\sqrt{\frac{216W}{35L} }[/tex]

Step-by-step explanation:

From the given question, the expression showing the relationship among the weight, length and diameter of the metal bar is;

W [tex]\alpha[/tex] L[tex]D^{2}[/tex]

W = kL[tex]D^{2}[/tex]

where k is the constant of proportionality.

When W = 140, D = 4 and L = 54, then;

140 = k(54)[tex](4)^{2}[/tex]

      = 864k

k = [tex]\frac{140}{864}[/tex]

  = [tex]\frac{35}{216}[/tex]

k = [tex]\frac{35}{216}[/tex]

⇒ W = [tex]\frac{35LD^{2} }{216}[/tex]

So that;

35L[tex]D^{2}[/tex] = 216W

[tex]D^{2}[/tex] = [tex]\frac{216W}{35L}[/tex]

D = [tex]\sqrt{\frac{216W}{35L} }[/tex]

Which is equivalent to (square root 10) ^3/4? ​

Answers

Answer:

2nd option

Step-by-step explanation:

Using the rule of exponents/ radicals

[tex]a^{\frac{m}{n} }[/tex] ⇔ [tex](\sqrt[n]{a}) ^{m}[/tex] , then

[tex]\sqrt{10} ^{\frac{3x}{4} }[/tex]

= [tex](\sqrt[4]{10}) ^{3x}[/tex]

Otto used 6 cups of whole wheat flour and x cups of white flour in the recipe. What is the equation that can be used
to find the value of y, the total amount of flour that Otto used in the recipe, and what are the constraints on the
values of x and y?
y=6x, x is any integer greater than or equal to 0, and y is an integer greater than or equal to 6.
O y=6x, x is any real number greater than or equal to 0, and y is any real number greater than or equal to 6.
y=x+6; x is any integer greater than or equal to 0, and y is an integer greater than or equal to 6.
O y=x+6; x is any real number greater than or equal to 0, and y is any real number greater than or equal to 6.

Answers

Answer:

y = x + 6

x is any real number greater than or equal to 0,

and y is any real number greater than or equal to 6.

The formula to determine the value of y, or the total amount of flour that Otto used in the recipe, is y=6x.

What is algebraic Expression?

Any mathematical statement that includes numbers, variables, and an arithmetic operation between them is known as an expression or algebraic expression. In the phrase 4m + 5, for instance, the terms 4m and 5 are separated from the variable m by the arithmetic sign +.

The correct equation is y = 6 + x, where y is the total amount of flour and x is the amount of white flour used.

The constraints on the values of x and y are x ≥ 0 (because you cannot use negative flour) and y ≥ 6 (because the minimum amount of flour used is 6 cups, and that is when x = 0).

Any real number more than or equal to 0 and any real number higher than or equal to 6 are the restrictions on the values of x and y, respectively.

So the correct answer is: y = x + 6; x is any real number greater than or equal to 0, and y is any real number greater than or equal to 6.

Learn more about algebraic Expression here:

https://brainly.com/question/953809

#SPJ7

Choose which group of measures would go along with the data set below.

11,13,11,9,8,4,7,7,11,9

1.
Range—4
Mode—8
Median—9
Mean—9


2.
Range—5
Mode—9
Median—7
Mean—7


3.
Range—9
Mode—11
Median—9
Mean—9
4.
Range—12
Mode—8
Median—9
Mean—9

Answers

3 because mode is 11 (11 is the number used the most) and the range is 9 (13-4=9)

Solve the following system of equations.
\begin{aligned} -5x+4y &= 3\\\\ x&=2y-15 \end{aligned}
−5x+4y
x


=3
=2y−15

x=?
y=?

Answers

Step-by-step explanation:

The first president of the United States was

[tex]\begin{aligned} -5x+4y &= 3\\\\ x&=2y-15 \end{aligned}[/tex]

Answer:

x=9

y=12

Step-by-step explanation:

Can someone help with this and help me understand this as well?

Answers

Answer:

<K ≅ <N

Step-by-step explanation:

JKL ≅ MNP

That means the angles are the same

<J ≅< M

<K ≅ <N

<L ≅<P

The sides are also congruent

JK ≅ MN

KL ≅ NP

JL ≅ MP

A person deposited Rs. 80,000 in bank 'P' for 2 years at the rate of 10% annual compound interest. But after one year bank has changed the policy and decided to pay semi-annual compound interest at the same rate. What is the percentage difference between compound interests of the first year and second year? Give reason with calculation:​

Answers

Answer:

compound interest in year 2 is 12.75% than compound interest in year 1. This is because semi annual compounding yield a higher compound interest

Step-by-step explanation:

compound interest = future value - present value

The formula for calculating future value:

FV = P (1 + r/m)^nm

FV = Future value  

P = Present value  

R = interest rate  

N = number of years

m = number of compounding  

compound value in the first year = 80,000(1.1)^1 = 88,000

compound interest = 88,000 - 80,000 = 8,000

compound interest in the second year = 88,000(1 + 0.01/2)^2 = 97,020

compound interest = 97,020 - 88,000 = 9020

Percentage change = (9020 / 8,000) - 1 = 12.75%

The three angles in a triangle have measures of 55°, 75°, and (2x)°. What is the value of x?

Answers

Answer:

x = 25

Step-by-step explanation:

The angles in a triangle add up to 180°, so we can write an equation to model the situation:

55 + 75 + 2x = 180

Now, we can simplify and solve:

130 + 2x = 180

2x = 50

x = 25

Answer:

x = 25

Step-by-step explanation:

The angles in a triangle add up to 180 degrees.

Using this, create an equation:

55 + 75 + 2x = 180

Solve for x:

55 + 75 + 2x = 180

130 + 2x = 180

2x = 50

x = 25

So, x = 25

What is the area of the polygon given below?

Answers

ANSWER:

B. 525 square units

SOLUTION:

S= 27+14+9×14+3×7

S= 525

[tex]{{hope it helps}}}[/tex]

Answer:

Option B 525 Square units

Step-by-step explanation:

7x3=21

27x14=378

14x9=126

21+378+126=525

One piece of wood has a rectangular bottom that is 30 cm long and 56 mm wide. The block is said to weigh 80 N. Calculate the pressure that the block exerts on the surface

Answers

Answer:

4762 N/m^2

Step-by-step explanation:

Find area.

30x5.6=168

Convert to m^2

168cm^2=0.0168m^2

Use pressure formula to get answer

P=F/A

P=80/0.0168

P=4762 N/m^2

16 is what percent less than 489?

Answers

hope this helps. Please mark me brainliest

Answer:

3.27

16 is what percent of 489? = 3.27.

a freight elevator has a weight limit of 2 tons. Each crate that is loaded weighs 80 pounds. What is the greatest number of crates that can be loaded on to the elevator?

25

500

250

50

Please help meh TvT

Answers

Answer:

50 crates

Step-by-step explanation:

First we need to convert 2 tons into pounds; 2 short tons = 4000 pounds, so the greatest number of crates that can be loaded onto the elevator is 4000 Hope this helps >:D

50 crates because u need to do a lot of things then u get it

Which are solutions to the system of equations? Select all that apply

{y=4x
y=4x^2+4x-1

A (1/2,2)
B.(-1/4,-1)
C. (-1/2,-2)
D. (1/4,1)

Answers

Answer:

A, C are both solutions

Step-by-step explanation:

B will not work because:

-1 = 4(-1/4)² + 4(-1/4) - 1

-1 = 1/4 - 1 - 1

-1 ≠ -1 3/4

D will not work because:

1 = 4(1/4)² + 4(1/4) - 1

1 = 1/4 + 1 - 1

1 ≠ 1/4

As an estimation we are told £3 is €4.
Convert €28 to pounds.

Answers

Hope this helps

Answer:

£21

€28 = £21

Please give real answers with explaination. 50 points + I will give brainliest. No Docs/No Files/No Links only answer with explaination.

Answers

Answer:

Step-by-step explanation:

That exterior angle is equal to the sum of the remote interiors, specifically:

2x + 3 = 45 + x + 8 and

x = 50

So the angles inside the triangle...we already know one of them is 45. If x = 50, then x + 8 is 50 + 8 = 58. That means that the third angle is 180 - 45 - 58 which is 77.

In the diagram below, AB = BC, and mZA = 69º. Find mZBCD.

Answers

Answer:

∠ BCD = 111°

Step-by-step explanation:

Since AB = BC then the triangle is isosceles and the 2 base angles are congruent.

∠ ACB = ∠ A = 69°

∠ ACB and ∠ BCD are adjacent angles and sum to 180° , that is

∠ BCD + ∠ ACB = 180°

∠ BCD + 69° = 180° ( subtract 69° from both sides )

∠ BCD = 111°

Pls help

An airplane can Seat up to 175 passengers the airplane has already sold 87 tickets for a flight on the airplane which graph represents the solution to the inequality that find a number of tickets the air line can still sell

Answers

Answer:

the third graph is the answer

Answer:

It's A.

Step-by-step explanation:

Since 175-87 is 88, there is 88 or less available tickets left. So it is A.

Hope this helps.

Other Questions
Match the metric measurement on the left with an equivalent unit of measurement on the right Write and Graph Inequalities 1. When cooking a turkey, the recommended internal temperature of the turkey is 165 F. A lower internal temperature will undercook the turkey. Write an inequality that is true only for temperatures that which the turkey will be undercooked.I want the answer in words pls and thank u What is the answer for earning in dollars A. 200B.150C.110D.120 High altitude adaptation is a complex physiological process that involves an increase in both the number of Hb molecules per erythrocyte and the total number of erythrocytes. It normally requires several weeks to complete. However, even after one day at high altitude, there is a significant degree of adaptation. This effect results from a rapid increase in the erythrocyte BPG concentration. a. Try to do this without looking at the graphs. If oxygen-binding curves for both high altitude adapted hemoglobin and normal, unadapted Hb were plotted together, would the curve for high altitude adapted Hb be to the left of, to the right of or the same as the curve for unadapted Hb? Explain. b. Is the oxygen binding affinity of high-altitude adapted Hb higher, lower, or the same as that of unadapted Hb? Explain. If a web address is written as http://www.mywebprescence.com/index.html: What part of this address is considered to be the webpage? The perimeter of a parallelogram must be no less than 40 feet. The length of the rectangle is 6 feet. What are the possible measurements of the width? Write an inequality to represent this problem. Use w to represent the width of the parallelogram. [Hint: The formula for finding the perimeter of a parallelogram is P = 2 l + 2 w . What is the smallest possible measurement of the width? Justify your answer by showing all your work. Read the excerpt from chapter 23 of The Adventures of Huckleberry Finn."Don't it s'prise you de way dem kings carries on, Huck?""No," I says, "it don't.""Why don't it, Huck?""Well, it don't, because it's in the breed. I reckon they're all alike,""But, Huck, dese kings o' ourn is reglar rapscallions; dat's jist what dey is; dey's reglar rapscallions.""Well, that's what I'm a-saying; all kings is mostly rapscallions, as fur as I can make out."Which best describes the satirical element of this excerpt?Jim and Huck still genuinely believe that therapscallions are royalty.Jim is revealing to Huck that most people are not what they seem.Jim tells Huck that he knew the truth about the kings before everyone else.Jim and Huck are basing their opinion of all royalty on fake kings. in order for the parallelogram to be a rhombus x= Who is a famous Greek athlete that increased his strength by lifting a small calf several times a week!A. AchillesB. CopernicusArchimedesC. Milo of Corona A bag contains 6 red balls and 5 black balls . Two balls are picked , one after the other . Find the probability that :i. Both are blackii. The first is blackiii. Both are of the same colour Given that :a. it is without replacement b. it is with replacement An 18-year-old primigravid woman comes for her initial prenatal visit at 16 weeks' gestation. She is not sure about the date of her last menstrual period but says that the pregnancy probably occurred immediately after she stopped taking oral contraceptives 5 months ago. Maternal serum -fetoprotein (MSAFP) level is increased to 3 multiples of the median. Which of the following is the most appropriate next step in management?a. Repeat measurement of MSAFP levelb. Triple screening for MSAFP, serum -hCG, and serum estriol levelsc. Ultrasonographyd. Amniocentesis for measurement of -fetoprotein levele. Amniocentesis for chromosomal analysis lolLloleoreniiwkv ibyuerw Help me plz I need to get a good score Containers of ice cream are sold by volume, not by weight. Some ice cream manufacturers pump air into the ice cream before packaging it. Why does this practice make the ice cream more expensive to the consumer? Equation of lines acellus pls help ofooehhenxkdoke critically discuss three nagative reosons why some people join protest action Explain the importance of a cross-cultural understanding for international businesses success. Seema is travelling to California. She travels a distance of 7,800 km by plane to Lima and then another 6,700 km to Los Angeles. Both planes fly at an average speed of 900 kmh-1.What is her total flight time in hours and minutes, to the nearest minute? A rectangular Carrer has a perimeter of 240cm breadth of 50cm.What is it's length EASY!! Please help!! 25 points An entrepreneur wants to start a new business. She needs a huge amount ofcapital to get started but is willing to divide the profits and control over thecompany among a large group of investors. Which type of business wouldbest suit her needs?A. Sole proprietorshipB. PartnershipC. Limited liability companyO D. Corporation